Вы находитесь на странице: 1из 14

Some Own Problems In Number Theory

Mathmdmb April 16, 2011


Here are some problems proposed by me, and the problems or their solutions have not been approved by someone else. So if any fault occurs, I shall take the whole responsibility. In this case, please inform me. Among the problems,many were posted by me on AoPS fora, so I thank the users who posted replies and solutions there. A notable fact is that I put the problems not in order to diculty, just randomly - which is, in my opinion, more interesting.

Notations
N = {1, 2, 3, . . . , n, . . .} : the set of positive integers. Z = {1, 2, 3, . . . , n, . . .} : the set of integers. N0 = {0, 1, 2, .....} : the set of nonnegative integers. a A : a is an element of the set A. a | b : b is divisible by a. a b : b is not divisible by a. a | b c : b and c are both divisible by a. gcd(a, b) : the Greatest Common Divisor of a and b. a and b are coprime : gcd(a, b) = 1. (m) : the number of positive integers x, not exceeding m with the property gcd(x, m) = 1.
n i=1

ai = a1 a2 ....an .

x : the largest integer not exceeding x. x : for all x. a is squarefree : there does not exist x N such that x2 | a. a N is a perfect number : sum of positive divisors of a is 2a. 1

ordm (a) = x : x is the order of a modulo m. vp (x) = : is the greatest positive integer such that p |x W 5 : Which is What We Wanted. W LOG : Without Loss Of Generality.

Problems

1. Prove that there does not exist a pair(n, m) N so that n+3m and n2 +3m2 are both perfect cubes. Find all such pairs if (m, n) Z. 2. Find all primes p such that the number 11p + 10p is a perfect power of a positive integer. 3. Let Fn = 22 + 1 be the nth Fermat number. Prove that 22
m n

+2n

F | Fn m 1 1,

m, n N.

4. Let a > 2 be an integer. Show that aa a is never square-free. 5. Show that there are innitely many pairs of positive integers (a, b) such that a5 5 if the number a3 b+b is an integer, then it is a perfect 5th power. 3 +1 6. Let p 2 (mod 3) be a prime number. Show that there exists a complete set of residue class of p such that the sum of its elements is divisible by p2 . 7. Prove that 81|10n+1 10 9n for all n N0 . 8. Find all positive integers n such that n|2n! 1. 9. Let p be a prime. Determine all perfect numbers having p factors. 10. Prove that a number which has only one prime factor, cant be a perfect number. 11. Solve in positive integers the equation a7 + b7 = 823543 (ac)1995 . 12. Find all n N such that 1. n2 27n + 182 is a perfect square. 2. n2 27n + 183 is a perfect square 13. Find all (a, b) N0 such that the number 7a + 11b is a perfect square. 14. Consider a complete set of residues modulo p. Show that we can partition this set into two subsets with equal number of elements such that the sum of elements in each set is divisible by p. 15. Let ai , m, and n be positive integers such that ai + m is a prime for all n 1 i n. Let N = i=1 pai and let S be the number of ways of expressing N i as a product of m positive integers. Prove that mn | S. 16. Find all integers (a, b, c, d) such that abc d = 1, bcd a = 2 3

17. Find all positive integer values that integers. 18. Solve in positive integers :

x2 +y 2 +1 xy+1

can take where x and y positive

n10 + n10 + .... + n10 = 19488391 1 2 8 19. Find all n such that (N )|N .

Solutions
(n + 3m)(n2 + 3m2 ) = a3 = n3 + 3m2 n + 3mn2 + 9m3 = a3 = (m + n)3 + (2m)3 = a3

1. If n + 3m, n2 + 3m2 both are cubes,then their product too. Then

By Fermats Last Theorem, the last equation has no solution. If (m, n) Z, then we have two cases: 1. m+n = 0 Then we get n+3m = 2m is a perfect cube which gives m = 2x3 . Also n2 + 3m2 = 4m2 From this we conclude that m = 4t3 , n = 4t3 . 2. 2m = 0 or m = 0 In this case, n + m = n = u3 and n2 + 3m2 = n2 = (u2 )3 Thus, (m, n) = (4t3 , 4t3 ), (0, u3 ) are only solutions. 2. First Solution : First look at p = 2,which is not a solution. Now p odd, so 11p + 10p is divisible by 11 + 10 = 21.So 3, 7|11p + 10p According to the Lifting The Exponent Lemma(LTE), we get that 32 |11p + 10p or 72 |11p + 10p if p = 3 or p = 7.Checking, we easily get p = 3, 7. So, it cant be a perfect power. Second Solution : Again p = 2.So 11p + 10p 0 (mod 9) The latter implies 2p 1 (mod 9), since 26n+3 1 (mod 9) it follows that 3|p, p = 3. But, 113 + 103 = 1331 + 1000 = 2331 is not a perfect power. Third Solution : First check with p = 2, 3 manually which dont work. So let p 5. We know that every prime greater than 3 is of the form 6k 1. So, two cases: 1. p = 6k + 1,then 11p + 10p 2p + 1 (26 )k .2 + 1 3 (mod 9).But any integer of the form 9m + 3 = 3(3m + 1) is not a perfect power because 9 does not divide it. 2. p = 6k +5, then 2p +1 25 +1 6 (mod 9) Since it is of the form 9m3, a similar logic works here too. Therefore, no solution exists.

3. First Solution : 2 2 From LTE, if v2 ( a b ) = , v2 (n) = ,then v2 (an bn ) = + 2 r Applying this,we get that,v2 (Fm 1) = v2 (Fm 1) + v2 (r) m Since v2 (Fm 1) = 2 , v2 (Fn 1) = 2n , now it follows that
F v2 (Fmn 1 1) = 2m + 2n

Then,22

+2n

F | Fmn 1 1

Second Solution : In this solution, we use the repeated use of the identity a2 1 = (a + 1)(a 1). Then, k k1 k1 k1 k2 k2 a2 1 = (a2 + 1)(a2 ) = (a2 + 1)(a2 + 1)((a2 1) = k1 k2 2 2 (a + 1)(a + 1).........(a + 1)(a 1) k This equation says that v2 (a2 1) = v2 (a 1) + v2 (2k ) = v2 (a 1) + k. n F Applying the result,it easily follows that v2 (Fmn 1 1) = v2 (Fm 1)+v2 (22 ) = 2m + 2 n .
F Note. Not only 22 +2 |Fmn 1 1, also 22 2m +2n Fm 1 1. Also 2 |Fn
m n m

+2n

F mostly divides Fmn 1 1.

4. aa a = a(aa1 1) and we claim that aa1 1 must have a square factor. Lets eliminate the trivial case a = 3 rst, since then 32 1 = 23 So a > 3.If a 1 has no odd prime factor, then a 1 must be a power of 2.Since a > 3, a 1 22 and thus it has a square factor. Assume to the contrary that, a 1 has an odd prime factor, say p. Then from the LTE, we nd that vp (aa1 1) is at least 2, implying that p2 |aa1 1, as desired. 5. This seems to be like the famous IMO-1988,6 which stated that if a +b is ab+1 an integer, then it is a perfect square.But this is not that hard at all. See, we are asked to prove the existence of such an innite couple,not to prove that,if a5 +b5 th power. This suggests us to nd a a3 b3 +1 is an integer,then it is a perfect 5 suitable parameter for a, b such that the condition satises. Lets recall the parameters of IMO-1988,6 where, we could nd an innite couples of a, b with a = c, b = c3 . Then, our intution says to choose a = bk The problem is reduced to nd k such that b5k5 + 1 = b3k+3 + 1 which gives k = 4. Indeed, a5 + b5 = b5 a3 b3 + 1 Thus, (a, b) = (c4 , c), (c, c4 ) gives such an innite couple. W 5
2 2

6. First we prove the following fact. Lemma : If a2 + ab + b2 is divisible by p 2 mod 3, then p|a, p|b Proof : Let p = 3k + 2. So, a2 + ab + b2 0 mod p = a3 b3 (mod p) Now from Fermats Little Theorem ap1 bp1 (mod p) It is well-known that if am bm (mod p) and an bn (mod p) then, agcd(m,n) bgcd(m,n) (mod p) = agcd(3,p1) bgcd(3,p1) (mod p) = agcd(3,3k+1) bgcd(3,3k+1) (mod p) = a b (mod p) = p|a b From this, we see that p|a2 2ab + b2 , a2 + ab + b2 and we can say p|a2 + ab + b2 (a2 2ab + b2 ) = 3ab Since gcd(p, 3) = 1, p|ab implying that p|a, p|b. Now back to the original problem. Lets consider the complete residue class 1, 2, ....., p 1.Then according to the lemma, 13 , 23 , ....., (p 1)3 is also a complete residue class of p.If two of its elements were congruent modulo p. Let i3 j 3 mod p = (ij)(i2 +ij +j 2 ) 0 mod p. Because i j < p, gcd(i j, p) = 1 and so i2 + ij + j 2 0 mod p which would imply p|i, j which is clearly absurd. Hence, any two terms are incongruent modulo p and it forms a complete residue 2 2 class. And the sum of them is p (p1) , it is obviously divisible by p2 . 4 7. First Solution : We use induction on n. The base case n0 = 0 is true since 81|101 10 0 = 0 Let it be true for some n.Then we require to prove that 81|10n+2 9n 19 10n+2 9n 19 = 10(10n+1 10 9n) + 81(n + 1) This is now obviously divisible by 81. Second Solution : This is solution is more elegant than the previous one. Consider the sum, S = 1 + 11 + 111 + ..... upto nth terms containing n 1 s. Lets try to nd a closed formula for S. 9S = 9 + 99 + 999 + ....... = 10 + 102 + 103 + ..... 1 1 1 ... = 10(1 + 101 + n 102 + ....) n = 10( 10 91 ) n n+1 So, S = 10 109n . 81 Since, S is a positive integer, 81 must divide 10n+1 10 9n.

8. Firstly, n odd. Now, 2(n) 1 (mod n) since gcd(n, 2) = 1. But, (n) < n, so n! must contain (n) as a factor since n! is the product of integers from 1 to n. Thus, (n)|n!. 2n! 1 mod n,so true for all n odd. 9. We claim that, if N has p factors, then N must be a perfect power of only one prime. This is because if N = pr ps with p1 , p2 distinct primes, then the number of 1 2 divisors of N is (r + 1)(s + 1) cant be a prime unless r = 0 or s = 0. Because, otherwise the number of divisors of N = (r + 1)(s + 1) which is not a prime. So, our claim is true. If the sum of proper divisors is equal to N ,say N = q p1 .Thus, 1 + q + q 2 + ...q p2 = N =
p1

q p1 1 = q p1 q1

1 But, it is not possible as q q1 < q p1 . This proves the desired fact.

10. This problem is actually a special case of the previous one. Note. In fact, we can say more specically that the sum of some divisors of a prime power or a number having a prime number of divisors. 11. This is an equation involving very high exponents. So, it cant have many solutions. In fact, we shall prove that it has no positive integer solution. First Solution : At rst note that 7|1995 and the consequence is obvious,we need to search for Fermats Last Theorem. So, we need to express 823543 as a perfect 7th power. Lets try a bit. The given number is not divisible by 3 because the sum of its digits 29 is not divisible by 3.So see if it is divisible by the next prime 7. A number is divisible by 7 i the number obtained by the subtraction of the three digit number formed by its last three digit and the number that is left. Example. 2401 is divisible by 7.See that, 401 2 = 399.Since,399 is divisible by 7,the original number is divisible by 7. Now, lets check whether 823543 is divisible by 7 or not.823 543 = 280 which is divisible by 7. This leads us to factorize 823543.Note that 823543 = 7 117649 Again,649117 = 532 = 776.Thus, 823543 = 72 16807 = 73 2401 = 74 73 = 77 The initial equation becomes a7 + b7 = (7a285 c285 )7 having no solution in positive integers. 8

Second Solution : This is an easier and more elementary,elegant one. Again,since a7 |a1995 , a7 |b7 . Let b = ak. Th equation stands k 7 + 1 = (7a284 c285) But, no two perfect 7th powers dont dier by 1. 12. First Solution : We shall use two very useful fact in solving this problem. It is a widely known tactic. There is no positive integer between two consecutive positive integers. So there is no perfect square which lies between two consecutive perfect squares. The product of two consecutive integers is not a perfect square. 1. Just note that n2 27n + 182 = (n 13)(n 14). Since n 13 and n 14 does not share any common factor,n13 and n14 both must be squares. But then the dierence of two squares is 1,contradiction! 2. Again, n2 27n + 183 = (n 13)(n 14) + 1. Let n 14 = m giving us m(m + 1)) + 1 = x2 If m is a positive integer, m2 < m2 + m + 1 < m2 + 2m + 1 = (m + 1)2 . If m is a negative integer, m2 > m2 + m + 1 > m2 + 2m + 1 Thus, n2 27n+183 cant be a perfect square except 1 yielding n = 14, 13.
7

Second Solution : This time we shall use discriminant of a quadratic equation . 1. Let, n2 27n + 182 = z 2 . Then, its discriminant is 272 4(182 z 2 ) = 4z 2 + 1 is a perfect square. But (2z)2 + 1 is a square only for z = 0. Then n = 13, 14.

2. The discriminant of n2 27n + 183 = z 2 is 729 4(183 z 2 ) = 4z 2 3. Let, 4z 2 3 = x2 = (2z + x)(2z x) = 3 = 2z + x = 3, 2z x = 1 = z = x = 1 which gives us n = 13, 14. 13. Let 7a + 11b = x2 . Lets make several cases : Case 1 : a = 0, b > 0. Then, the equation is reduced to 11b = (x + 1)(x 1) which means that 11|x + 1 x 1 = 11|2 Contradiction ! Case 2 : b = 0, a > 0. This is similar to the preceeding one. Case 3 : a, b > 0. Then 7a + 11b must be even. So 7a + 11b 0 (mod 4). Take modulo 4 in the equation. So, x2 0 (mod 4) = 7a + 11b 0 (mod 4) = (1)a + (1)b 0 (mod 4) From the latter,we may conclude that a and b are of dierent pairity. Otherwise x2 2 (mod 4) which is not true. Again, take modulo 3. If y even, 7a + 11b 2 (mod 3) but no square gives remainder 2 upon division by 3. This makes b odd and a even. Let a = 2k. Thus, 11b = (x + 7k )(x 7k ) = 11|x + 7k x 7k

= 11|2 7k Contradiction ! The only case left is b = 1 which does not produce any solution. Therefore, no such a, b exists. 14. Lemma : If p > 3 a prime, then 24|p2 1. Proof : Since p > 3, obviously p odd and not divisible by 3. Thus p 1 (mod 3) and p 1 (mod 2). Combining theses two p 1 (mod 6). Let, p = 6k 1 = p2 = 36k 2 12k + 1 = 12k(3k + 1) = p2 1 10

k and 3k + 1 are of opposite pairity since their dierence is not divisible by 2. So, one of them must be even and thus, 122 = 24|p2 1. The complete residue class is {1, 2, ......, p 1} Now, recall that there are exactly p1 quadratic residues mod p. The sum of 2 the residues is p(p1) which is divisible by p. 2 So, we seek for only a set with p1 elements, since then the sum of the elements 2 of the other set will be divisible by p automatically. And consider the quadratic residues 12 , 22 , ....., ( p1 )2 . 2 Note that, their sum is 12 + 22 + ... + ( Now, we are done if 24|p2 1 W 5 . 15. Lemma 1 : For p prime and a positive integer r < p, r| p1 . r1 Proof : Note the identity : p p1 p = r r r1 Here, gcd(r, p) = 1 but p is a positive integer. So r must divide p1 . r r1 Lemma 2 : n If N = i=1 pai and the number of ways to write N as a product of m positive i integers, then n ai + m 1 S= m1 i=1 Proof : +m1 Take any ai . From the balls in urns principle, there are aim1 ways. n n ai +m1 ai +m1 Thus, there are i=1 m1 ways and S = i=1 m1 . qi 1 Let, ai + m = qi where qi are primes. Then, from lemma 1 above, m| m1 for all qi . Multiplying them together we get,
n

p(p2 1) p1 2 ) = 2 24

mn |
i=1

ai + m 1 m1

=S

16. From the equations, abcd = d2 + d and abcd = a2 + 2a implying that d2 + d + 1 = (a + 1)2 If d N, d2 + d + 1 is not a perfect square from problem 12. So, we have 1 d 0. If d = 0, a = 2 and abc = 1 = 2bc = 1, no solution. 11

If d = 1, abc = 0 and bc = (a + 2). Now, a(a + 2) = d(d + 1) = 0 giving a = 0 or a = 2. If a = 0, bc = 2 = (b, c) = (1, 2), (1, 2), (2, 1), (2, 1) 17. We can use the famous Vietta Jumping to solve this problem. We claim 2 +y 2 +1 that (x, y) = (c, c+1) is the only pair for which the fraction x xy+1 is a positive integer and thus the only integer value it can assume is 2. Let x2 + y 2 + 1 =k xy + 1

and the smallest solution of be (x, y). Since is symmetric over x, y we can assume W LOG x y. Note that x = y yields x2 + 1|2x2 + 1 2x2 + 2 = x2 + 1|1 < 2 x2 + 1, contradiction !! Therefore, x > y. x2 kxy + y 2 + 1 k = 0 Fix in y, k and consider all values of (x, y). will be quadratic in x and let the other root of be x1 . From Viettas formula, x + x1 = ky xx1 = y 2 + 1 k Thus x1 = y2 y2 y2 + 1 k < < =y x x y

But then (x, x1 ) < (x, y) would be a smaller solution than the smallest solution. Contradiction ! 18. From Fermats little theorem, a111 = a10 0 or 1 (mod 11). n10 + n10 + ... + n10 0, 1, 2, 3, 4, 5, 6, 7, 8 1 2 8 (mod 11)

But 19488391 10 (mod 11) which isnt possible by any combination of the above. 19. Let, the canonical prime factorization of N is, N = pa1 ....pan Then it is n 1 well known that (N ) = pa1 1 ....pan 1 .(p1 1)...(pn 1) n 1 . Therefore, (N )|N implies (p1 1)....(pn 1)|p1 ...pn . So, one of the primes, say p1 = 2. Moreover, N cant have more than two prime factors, otherwise 2p2 ...pn would be divisible by 4, but p2 , ..., pn are distinct odd prime factors. This yields a contradiction ! Now we have N = 2a pb . Then p 1|2p. Since Gcd(p 1, p) = 1, they can not share any common factor. So, p 1|2 and because p odd, we conclude that 12

p = 3. Thus, we found an innite numbers satisfying this property with N = 2a 3b . Note that a is necessarily positive integer. Note. We can re-state the problem as : Find all N such that there are N k positive numbers less than N and co-prime to N . Then the value of k would be 3.

13

References
[1] AoPS topic #393335, Lifting The Exponent Lemma (Containing PDF le), posted by Amir Hossein Parvardi (amparvardi on AoPS). http://www.artofproblemsolving.com/Forum/viewtopic.php?t=393335 [2] Santiago Cuellar, Jose Alejandro Samper, A nice and tricky lemma (lifting the exponent), Mathematical Reections 3 2007. [3] AoPS topic #356847, n+3 perfect cube (Problem 1 in this article), posted by mathmdmb. http://www.artofproblemsolving.com/Forum/viewtopic.php?t=356847 [4] AoPS topic #36224, perfect power (Problem 2 in this article), posted by mathmdmb. http://www.artofproblemsolving.com/Forum/viewtopic.php?t=362224 [5] AoPS topic #393335,Lifting The Exponent Lemma (Problem 5 in this article), posted by amparvardi. http://www.artofproblemsolving.com/Forum/viewtopic.php?p=2198886 [6] AoPS blog #85314,The Law Of Nature-Number Theory (Problem 8 in this article), posted by mathmdmb. http://www.artofproblemsolving.com/Forum/blog.php?u=85314 [7] AoPS topic #356079, Find all n (Problem 16 in this article), posted by mathmdmb. http://www.artofproblemsolving.com/Forum/viewtopic.php?t=356079 [8] AoPS topic #304361, Problem 202 in Number Theory Marathon (problem 17 in this article), posted by mathmdmb. http://www.artofproblemsolving.com/Forum/viewtopic.php?p=2223300 [9] AoPS topic #365172, Solve equation (problem in this article), posted by mathmdmb. http://www.artofproblemsolving.com/Forum/viewtopic.php?f=57t=365172hilit=solve+in+Z [9] AoPS topic #362222, equation (problem in this article),posted by mathmdmb. http://www.artofproblemsolving.com/Forum/viewtopic.php?f=57t=362222hilit=solve+in+Z

14

Вам также может понравиться